Integrate: $int_{-infty}^infty exp(-||vec x||^2) ||vec x||^{-m}mathrm{d}vec{x}$












0












$begingroup$


Let $m,n$ be two positive integers with $0 < m < n$.



Can we integrate this:



$$I = int_{-infty}^infty mathrm{d}x_1 dots int_{-infty}^infty mathrm{d}x_n left(sum_{i=1}^n x_i^2right)^{-m/2} expleft(-sum_{i=1}^n x_i^2right)$$



If a closed analytical expression is not possible, what I really need is to evaluate a large deviation limit of the form:



$$lim_{nrightarrowinfty} frac{1}{n} log I$$



where it is assumed that the ratio $m/n$ remains fixed.



Note that $I$ resembles a negative moment of the radious of a multivariate Gaussian.



I asked a very similar question here: Integrate: $int_0^1 ||vec x||^{-m}mathrm{d}vec{x}$. But note that this integral has different limits and a decaying quadratic exponential. Maybe I have better luck with this variant ;)










share|cite|improve this question











$endgroup$

















    0












    $begingroup$


    Let $m,n$ be two positive integers with $0 < m < n$.



    Can we integrate this:



    $$I = int_{-infty}^infty mathrm{d}x_1 dots int_{-infty}^infty mathrm{d}x_n left(sum_{i=1}^n x_i^2right)^{-m/2} expleft(-sum_{i=1}^n x_i^2right)$$



    If a closed analytical expression is not possible, what I really need is to evaluate a large deviation limit of the form:



    $$lim_{nrightarrowinfty} frac{1}{n} log I$$



    where it is assumed that the ratio $m/n$ remains fixed.



    Note that $I$ resembles a negative moment of the radious of a multivariate Gaussian.



    I asked a very similar question here: Integrate: $int_0^1 ||vec x||^{-m}mathrm{d}vec{x}$. But note that this integral has different limits and a decaying quadratic exponential. Maybe I have better luck with this variant ;)










    share|cite|improve this question











    $endgroup$















      0












      0








      0





      $begingroup$


      Let $m,n$ be two positive integers with $0 < m < n$.



      Can we integrate this:



      $$I = int_{-infty}^infty mathrm{d}x_1 dots int_{-infty}^infty mathrm{d}x_n left(sum_{i=1}^n x_i^2right)^{-m/2} expleft(-sum_{i=1}^n x_i^2right)$$



      If a closed analytical expression is not possible, what I really need is to evaluate a large deviation limit of the form:



      $$lim_{nrightarrowinfty} frac{1}{n} log I$$



      where it is assumed that the ratio $m/n$ remains fixed.



      Note that $I$ resembles a negative moment of the radious of a multivariate Gaussian.



      I asked a very similar question here: Integrate: $int_0^1 ||vec x||^{-m}mathrm{d}vec{x}$. But note that this integral has different limits and a decaying quadratic exponential. Maybe I have better luck with this variant ;)










      share|cite|improve this question











      $endgroup$




      Let $m,n$ be two positive integers with $0 < m < n$.



      Can we integrate this:



      $$I = int_{-infty}^infty mathrm{d}x_1 dots int_{-infty}^infty mathrm{d}x_n left(sum_{i=1}^n x_i^2right)^{-m/2} expleft(-sum_{i=1}^n x_i^2right)$$



      If a closed analytical expression is not possible, what I really need is to evaluate a large deviation limit of the form:



      $$lim_{nrightarrowinfty} frac{1}{n} log I$$



      where it is assumed that the ratio $m/n$ remains fixed.



      Note that $I$ resembles a negative moment of the radious of a multivariate Gaussian.



      I asked a very similar question here: Integrate: $int_0^1 ||vec x||^{-m}mathrm{d}vec{x}$. But note that this integral has different limits and a decaying quadratic exponential. Maybe I have better luck with this variant ;)







      normal-distribution large-deviation-theory






      share|cite|improve this question















      share|cite|improve this question













      share|cite|improve this question




      share|cite|improve this question








      edited Dec 6 '18 at 19:24







      becko

















      asked Dec 6 '18 at 19:20









      beckobecko

      2,34931942




      2,34931942






















          2 Answers
          2






          active

          oldest

          votes


















          1












          $begingroup$

          Consider the homomorphism $x mapsto left( |x|, dfrac{x}{|x|} right)$ between $mathbf{R}^d setminus {0}$ and $(0, infty) times mathbf{S}_{d-1}.$ By the change of measures formula, the integral becomes
          $$begin{align*}
          intlimits_0^infty dr intlimits_{mathbf{S}_{d-1}} dsigma_d e^{-r^2}r^{-m} r^{d-1} &= sigma_{d-1}(mathbf{S}_{d-1}) intlimits_{0}^infty dr e^{-r^2} r^{d-m-1} \
          &= dfrac{sigma_{d-1}(mathbf{S}_{d-1})}{2} intlimits_0^infty dr e^{-r} r^{frac{d-m-2}{2}} \
          &= dfrac{sigma_{d-1}(mathbf{S}_{d-1})}{2} Gammaleft(dfrac{d-m}{2}right). square
          end{align*}$$






          share|cite|improve this answer









          $endgroup$













          • $begingroup$
            Thanks, but the notation is a bit obscure to me. This is the same as what I got, right?
            $endgroup$
            – becko
            Dec 6 '18 at 20:40












          • $begingroup$
            Yours is wrong, I think. To begin with, $N = d$ and $M = m,$ and in the integral you made the change of variables $u = r^2,$ so that $u' = 2r$ and then $int e^{-r^2} r^alpha dr = dfrac{1}{2}int e^{-u} u^{frac{alpha - 1}{2}} du.$ I think you factored out a $2^{frac{d - m}{2}-1}$ wrongly.
            $endgroup$
            – Will M.
            Dec 7 '18 at 1:54










          • $begingroup$
            I see what's wrong. In the original question the exponent was not divided by 2, but in my answer for some reason I divided by 2. If you correct for that we are getting equivalent results.
            $endgroup$
            – becko
            Dec 7 '18 at 15:33



















          0












          $begingroup$

          Introduce the spherical coordinates $r, theta, xi_1, ldots, xi_{N - 2}$,
          $$ x_1 = r cos (xi_1), quad x_2 = r sin (xi_1) cos (xi_2), quad
          ldots, quad x_N = r sin (xi_1) ldots sin (xi_{N - 2}) sin
          (theta) $$

          where $0 leqslant xi_i leqslant pi$, $0 leqslant theta leqslant 2 pi$
          and $0 leqslant r < infty$. The volume element is given by $mathrm d x_1
          ldots mathrm d x_N = r^{N - 1} mathrm d r mathrm d sigma$
          , where $mathrm d
          sigma$
          is the unit-sphere surface element,



          $$mathrm d sigma = sin^{N - 2} (xi_1) ldots sin (xi_{n - 2}) mathrm d theta
          mathrm d xi_1 ldots mathrm d xi_{N - 2}$$



          It follows that:
          $$begin{eqnarray*}
          I & = & int_{- infty}^{infty} | {vec x} |^{- M} mathrm e^{- |
          {vec x} |^2 / 2} mathrm d vec x\
          & = & left( int_0^{infty} r^{N - M - 1} mathrm e^{- r^2/2}
          mathrm d r right) left( int_{mathcal{S}} mathrm d sigma right)\
          & = & 2^{(N - M) / 2 - 1} Gamma left( frac{N - M}{2}
          right) frac{2 pi^{N / 2}}{Gamma (N / 2)}
          end{eqnarray*}$$



          where $mathcal S$ is the unit sphere.






          share|cite|improve this answer









          $endgroup$













            Your Answer





            StackExchange.ifUsing("editor", function () {
            return StackExchange.using("mathjaxEditing", function () {
            StackExchange.MarkdownEditor.creationCallbacks.add(function (editor, postfix) {
            StackExchange.mathjaxEditing.prepareWmdForMathJax(editor, postfix, [["$", "$"], ["\\(","\\)"]]);
            });
            });
            }, "mathjax-editing");

            StackExchange.ready(function() {
            var channelOptions = {
            tags: "".split(" "),
            id: "69"
            };
            initTagRenderer("".split(" "), "".split(" "), channelOptions);

            StackExchange.using("externalEditor", function() {
            // Have to fire editor after snippets, if snippets enabled
            if (StackExchange.settings.snippets.snippetsEnabled) {
            StackExchange.using("snippets", function() {
            createEditor();
            });
            }
            else {
            createEditor();
            }
            });

            function createEditor() {
            StackExchange.prepareEditor({
            heartbeatType: 'answer',
            autoActivateHeartbeat: false,
            convertImagesToLinks: true,
            noModals: true,
            showLowRepImageUploadWarning: true,
            reputationToPostImages: 10,
            bindNavPrevention: true,
            postfix: "",
            imageUploader: {
            brandingHtml: "Powered by u003ca class="icon-imgur-white" href="https://imgur.com/"u003eu003c/au003e",
            contentPolicyHtml: "User contributions licensed under u003ca href="https://creativecommons.org/licenses/by-sa/3.0/"u003ecc by-sa 3.0 with attribution requiredu003c/au003e u003ca href="https://stackoverflow.com/legal/content-policy"u003e(content policy)u003c/au003e",
            allowUrls: true
            },
            noCode: true, onDemand: true,
            discardSelector: ".discard-answer"
            ,immediatelyShowMarkdownHelp:true
            });


            }
            });














            draft saved

            draft discarded


















            StackExchange.ready(
            function () {
            StackExchange.openid.initPostLogin('.new-post-login', 'https%3a%2f%2fmath.stackexchange.com%2fquestions%2f3028920%2fintegrate-int-infty-infty-exp-vec-x2-vec-x-m-mathrmd%23new-answer', 'question_page');
            }
            );

            Post as a guest















            Required, but never shown

























            2 Answers
            2






            active

            oldest

            votes








            2 Answers
            2






            active

            oldest

            votes









            active

            oldest

            votes






            active

            oldest

            votes









            1












            $begingroup$

            Consider the homomorphism $x mapsto left( |x|, dfrac{x}{|x|} right)$ between $mathbf{R}^d setminus {0}$ and $(0, infty) times mathbf{S}_{d-1}.$ By the change of measures formula, the integral becomes
            $$begin{align*}
            intlimits_0^infty dr intlimits_{mathbf{S}_{d-1}} dsigma_d e^{-r^2}r^{-m} r^{d-1} &= sigma_{d-1}(mathbf{S}_{d-1}) intlimits_{0}^infty dr e^{-r^2} r^{d-m-1} \
            &= dfrac{sigma_{d-1}(mathbf{S}_{d-1})}{2} intlimits_0^infty dr e^{-r} r^{frac{d-m-2}{2}} \
            &= dfrac{sigma_{d-1}(mathbf{S}_{d-1})}{2} Gammaleft(dfrac{d-m}{2}right). square
            end{align*}$$






            share|cite|improve this answer









            $endgroup$













            • $begingroup$
              Thanks, but the notation is a bit obscure to me. This is the same as what I got, right?
              $endgroup$
              – becko
              Dec 6 '18 at 20:40












            • $begingroup$
              Yours is wrong, I think. To begin with, $N = d$ and $M = m,$ and in the integral you made the change of variables $u = r^2,$ so that $u' = 2r$ and then $int e^{-r^2} r^alpha dr = dfrac{1}{2}int e^{-u} u^{frac{alpha - 1}{2}} du.$ I think you factored out a $2^{frac{d - m}{2}-1}$ wrongly.
              $endgroup$
              – Will M.
              Dec 7 '18 at 1:54










            • $begingroup$
              I see what's wrong. In the original question the exponent was not divided by 2, but in my answer for some reason I divided by 2. If you correct for that we are getting equivalent results.
              $endgroup$
              – becko
              Dec 7 '18 at 15:33
















            1












            $begingroup$

            Consider the homomorphism $x mapsto left( |x|, dfrac{x}{|x|} right)$ between $mathbf{R}^d setminus {0}$ and $(0, infty) times mathbf{S}_{d-1}.$ By the change of measures formula, the integral becomes
            $$begin{align*}
            intlimits_0^infty dr intlimits_{mathbf{S}_{d-1}} dsigma_d e^{-r^2}r^{-m} r^{d-1} &= sigma_{d-1}(mathbf{S}_{d-1}) intlimits_{0}^infty dr e^{-r^2} r^{d-m-1} \
            &= dfrac{sigma_{d-1}(mathbf{S}_{d-1})}{2} intlimits_0^infty dr e^{-r} r^{frac{d-m-2}{2}} \
            &= dfrac{sigma_{d-1}(mathbf{S}_{d-1})}{2} Gammaleft(dfrac{d-m}{2}right). square
            end{align*}$$






            share|cite|improve this answer









            $endgroup$













            • $begingroup$
              Thanks, but the notation is a bit obscure to me. This is the same as what I got, right?
              $endgroup$
              – becko
              Dec 6 '18 at 20:40












            • $begingroup$
              Yours is wrong, I think. To begin with, $N = d$ and $M = m,$ and in the integral you made the change of variables $u = r^2,$ so that $u' = 2r$ and then $int e^{-r^2} r^alpha dr = dfrac{1}{2}int e^{-u} u^{frac{alpha - 1}{2}} du.$ I think you factored out a $2^{frac{d - m}{2}-1}$ wrongly.
              $endgroup$
              – Will M.
              Dec 7 '18 at 1:54










            • $begingroup$
              I see what's wrong. In the original question the exponent was not divided by 2, but in my answer for some reason I divided by 2. If you correct for that we are getting equivalent results.
              $endgroup$
              – becko
              Dec 7 '18 at 15:33














            1












            1








            1





            $begingroup$

            Consider the homomorphism $x mapsto left( |x|, dfrac{x}{|x|} right)$ between $mathbf{R}^d setminus {0}$ and $(0, infty) times mathbf{S}_{d-1}.$ By the change of measures formula, the integral becomes
            $$begin{align*}
            intlimits_0^infty dr intlimits_{mathbf{S}_{d-1}} dsigma_d e^{-r^2}r^{-m} r^{d-1} &= sigma_{d-1}(mathbf{S}_{d-1}) intlimits_{0}^infty dr e^{-r^2} r^{d-m-1} \
            &= dfrac{sigma_{d-1}(mathbf{S}_{d-1})}{2} intlimits_0^infty dr e^{-r} r^{frac{d-m-2}{2}} \
            &= dfrac{sigma_{d-1}(mathbf{S}_{d-1})}{2} Gammaleft(dfrac{d-m}{2}right). square
            end{align*}$$






            share|cite|improve this answer









            $endgroup$



            Consider the homomorphism $x mapsto left( |x|, dfrac{x}{|x|} right)$ between $mathbf{R}^d setminus {0}$ and $(0, infty) times mathbf{S}_{d-1}.$ By the change of measures formula, the integral becomes
            $$begin{align*}
            intlimits_0^infty dr intlimits_{mathbf{S}_{d-1}} dsigma_d e^{-r^2}r^{-m} r^{d-1} &= sigma_{d-1}(mathbf{S}_{d-1}) intlimits_{0}^infty dr e^{-r^2} r^{d-m-1} \
            &= dfrac{sigma_{d-1}(mathbf{S}_{d-1})}{2} intlimits_0^infty dr e^{-r} r^{frac{d-m-2}{2}} \
            &= dfrac{sigma_{d-1}(mathbf{S}_{d-1})}{2} Gammaleft(dfrac{d-m}{2}right). square
            end{align*}$$







            share|cite|improve this answer












            share|cite|improve this answer



            share|cite|improve this answer










            answered Dec 6 '18 at 19:53









            Will M.Will M.

            2,440314




            2,440314












            • $begingroup$
              Thanks, but the notation is a bit obscure to me. This is the same as what I got, right?
              $endgroup$
              – becko
              Dec 6 '18 at 20:40












            • $begingroup$
              Yours is wrong, I think. To begin with, $N = d$ and $M = m,$ and in the integral you made the change of variables $u = r^2,$ so that $u' = 2r$ and then $int e^{-r^2} r^alpha dr = dfrac{1}{2}int e^{-u} u^{frac{alpha - 1}{2}} du.$ I think you factored out a $2^{frac{d - m}{2}-1}$ wrongly.
              $endgroup$
              – Will M.
              Dec 7 '18 at 1:54










            • $begingroup$
              I see what's wrong. In the original question the exponent was not divided by 2, but in my answer for some reason I divided by 2. If you correct for that we are getting equivalent results.
              $endgroup$
              – becko
              Dec 7 '18 at 15:33


















            • $begingroup$
              Thanks, but the notation is a bit obscure to me. This is the same as what I got, right?
              $endgroup$
              – becko
              Dec 6 '18 at 20:40












            • $begingroup$
              Yours is wrong, I think. To begin with, $N = d$ and $M = m,$ and in the integral you made the change of variables $u = r^2,$ so that $u' = 2r$ and then $int e^{-r^2} r^alpha dr = dfrac{1}{2}int e^{-u} u^{frac{alpha - 1}{2}} du.$ I think you factored out a $2^{frac{d - m}{2}-1}$ wrongly.
              $endgroup$
              – Will M.
              Dec 7 '18 at 1:54










            • $begingroup$
              I see what's wrong. In the original question the exponent was not divided by 2, but in my answer for some reason I divided by 2. If you correct for that we are getting equivalent results.
              $endgroup$
              – becko
              Dec 7 '18 at 15:33
















            $begingroup$
            Thanks, but the notation is a bit obscure to me. This is the same as what I got, right?
            $endgroup$
            – becko
            Dec 6 '18 at 20:40






            $begingroup$
            Thanks, but the notation is a bit obscure to me. This is the same as what I got, right?
            $endgroup$
            – becko
            Dec 6 '18 at 20:40














            $begingroup$
            Yours is wrong, I think. To begin with, $N = d$ and $M = m,$ and in the integral you made the change of variables $u = r^2,$ so that $u' = 2r$ and then $int e^{-r^2} r^alpha dr = dfrac{1}{2}int e^{-u} u^{frac{alpha - 1}{2}} du.$ I think you factored out a $2^{frac{d - m}{2}-1}$ wrongly.
            $endgroup$
            – Will M.
            Dec 7 '18 at 1:54




            $begingroup$
            Yours is wrong, I think. To begin with, $N = d$ and $M = m,$ and in the integral you made the change of variables $u = r^2,$ so that $u' = 2r$ and then $int e^{-r^2} r^alpha dr = dfrac{1}{2}int e^{-u} u^{frac{alpha - 1}{2}} du.$ I think you factored out a $2^{frac{d - m}{2}-1}$ wrongly.
            $endgroup$
            – Will M.
            Dec 7 '18 at 1:54












            $begingroup$
            I see what's wrong. In the original question the exponent was not divided by 2, but in my answer for some reason I divided by 2. If you correct for that we are getting equivalent results.
            $endgroup$
            – becko
            Dec 7 '18 at 15:33




            $begingroup$
            I see what's wrong. In the original question the exponent was not divided by 2, but in my answer for some reason I divided by 2. If you correct for that we are getting equivalent results.
            $endgroup$
            – becko
            Dec 7 '18 at 15:33











            0












            $begingroup$

            Introduce the spherical coordinates $r, theta, xi_1, ldots, xi_{N - 2}$,
            $$ x_1 = r cos (xi_1), quad x_2 = r sin (xi_1) cos (xi_2), quad
            ldots, quad x_N = r sin (xi_1) ldots sin (xi_{N - 2}) sin
            (theta) $$

            where $0 leqslant xi_i leqslant pi$, $0 leqslant theta leqslant 2 pi$
            and $0 leqslant r < infty$. The volume element is given by $mathrm d x_1
            ldots mathrm d x_N = r^{N - 1} mathrm d r mathrm d sigma$
            , where $mathrm d
            sigma$
            is the unit-sphere surface element,



            $$mathrm d sigma = sin^{N - 2} (xi_1) ldots sin (xi_{n - 2}) mathrm d theta
            mathrm d xi_1 ldots mathrm d xi_{N - 2}$$



            It follows that:
            $$begin{eqnarray*}
            I & = & int_{- infty}^{infty} | {vec x} |^{- M} mathrm e^{- |
            {vec x} |^2 / 2} mathrm d vec x\
            & = & left( int_0^{infty} r^{N - M - 1} mathrm e^{- r^2/2}
            mathrm d r right) left( int_{mathcal{S}} mathrm d sigma right)\
            & = & 2^{(N - M) / 2 - 1} Gamma left( frac{N - M}{2}
            right) frac{2 pi^{N / 2}}{Gamma (N / 2)}
            end{eqnarray*}$$



            where $mathcal S$ is the unit sphere.






            share|cite|improve this answer









            $endgroup$


















              0












              $begingroup$

              Introduce the spherical coordinates $r, theta, xi_1, ldots, xi_{N - 2}$,
              $$ x_1 = r cos (xi_1), quad x_2 = r sin (xi_1) cos (xi_2), quad
              ldots, quad x_N = r sin (xi_1) ldots sin (xi_{N - 2}) sin
              (theta) $$

              where $0 leqslant xi_i leqslant pi$, $0 leqslant theta leqslant 2 pi$
              and $0 leqslant r < infty$. The volume element is given by $mathrm d x_1
              ldots mathrm d x_N = r^{N - 1} mathrm d r mathrm d sigma$
              , where $mathrm d
              sigma$
              is the unit-sphere surface element,



              $$mathrm d sigma = sin^{N - 2} (xi_1) ldots sin (xi_{n - 2}) mathrm d theta
              mathrm d xi_1 ldots mathrm d xi_{N - 2}$$



              It follows that:
              $$begin{eqnarray*}
              I & = & int_{- infty}^{infty} | {vec x} |^{- M} mathrm e^{- |
              {vec x} |^2 / 2} mathrm d vec x\
              & = & left( int_0^{infty} r^{N - M - 1} mathrm e^{- r^2/2}
              mathrm d r right) left( int_{mathcal{S}} mathrm d sigma right)\
              & = & 2^{(N - M) / 2 - 1} Gamma left( frac{N - M}{2}
              right) frac{2 pi^{N / 2}}{Gamma (N / 2)}
              end{eqnarray*}$$



              where $mathcal S$ is the unit sphere.






              share|cite|improve this answer









              $endgroup$
















                0












                0








                0





                $begingroup$

                Introduce the spherical coordinates $r, theta, xi_1, ldots, xi_{N - 2}$,
                $$ x_1 = r cos (xi_1), quad x_2 = r sin (xi_1) cos (xi_2), quad
                ldots, quad x_N = r sin (xi_1) ldots sin (xi_{N - 2}) sin
                (theta) $$

                where $0 leqslant xi_i leqslant pi$, $0 leqslant theta leqslant 2 pi$
                and $0 leqslant r < infty$. The volume element is given by $mathrm d x_1
                ldots mathrm d x_N = r^{N - 1} mathrm d r mathrm d sigma$
                , where $mathrm d
                sigma$
                is the unit-sphere surface element,



                $$mathrm d sigma = sin^{N - 2} (xi_1) ldots sin (xi_{n - 2}) mathrm d theta
                mathrm d xi_1 ldots mathrm d xi_{N - 2}$$



                It follows that:
                $$begin{eqnarray*}
                I & = & int_{- infty}^{infty} | {vec x} |^{- M} mathrm e^{- |
                {vec x} |^2 / 2} mathrm d vec x\
                & = & left( int_0^{infty} r^{N - M - 1} mathrm e^{- r^2/2}
                mathrm d r right) left( int_{mathcal{S}} mathrm d sigma right)\
                & = & 2^{(N - M) / 2 - 1} Gamma left( frac{N - M}{2}
                right) frac{2 pi^{N / 2}}{Gamma (N / 2)}
                end{eqnarray*}$$



                where $mathcal S$ is the unit sphere.






                share|cite|improve this answer









                $endgroup$



                Introduce the spherical coordinates $r, theta, xi_1, ldots, xi_{N - 2}$,
                $$ x_1 = r cos (xi_1), quad x_2 = r sin (xi_1) cos (xi_2), quad
                ldots, quad x_N = r sin (xi_1) ldots sin (xi_{N - 2}) sin
                (theta) $$

                where $0 leqslant xi_i leqslant pi$, $0 leqslant theta leqslant 2 pi$
                and $0 leqslant r < infty$. The volume element is given by $mathrm d x_1
                ldots mathrm d x_N = r^{N - 1} mathrm d r mathrm d sigma$
                , where $mathrm d
                sigma$
                is the unit-sphere surface element,



                $$mathrm d sigma = sin^{N - 2} (xi_1) ldots sin (xi_{n - 2}) mathrm d theta
                mathrm d xi_1 ldots mathrm d xi_{N - 2}$$



                It follows that:
                $$begin{eqnarray*}
                I & = & int_{- infty}^{infty} | {vec x} |^{- M} mathrm e^{- |
                {vec x} |^2 / 2} mathrm d vec x\
                & = & left( int_0^{infty} r^{N - M - 1} mathrm e^{- r^2/2}
                mathrm d r right) left( int_{mathcal{S}} mathrm d sigma right)\
                & = & 2^{(N - M) / 2 - 1} Gamma left( frac{N - M}{2}
                right) frac{2 pi^{N / 2}}{Gamma (N / 2)}
                end{eqnarray*}$$



                where $mathcal S$ is the unit sphere.







                share|cite|improve this answer












                share|cite|improve this answer



                share|cite|improve this answer










                answered Dec 6 '18 at 19:24









                beckobecko

                2,34931942




                2,34931942






























                    draft saved

                    draft discarded




















































                    Thanks for contributing an answer to Mathematics Stack Exchange!


                    • Please be sure to answer the question. Provide details and share your research!

                    But avoid



                    • Asking for help, clarification, or responding to other answers.

                    • Making statements based on opinion; back them up with references or personal experience.


                    Use MathJax to format equations. MathJax reference.


                    To learn more, see our tips on writing great answers.




                    draft saved


                    draft discarded














                    StackExchange.ready(
                    function () {
                    StackExchange.openid.initPostLogin('.new-post-login', 'https%3a%2f%2fmath.stackexchange.com%2fquestions%2f3028920%2fintegrate-int-infty-infty-exp-vec-x2-vec-x-m-mathrmd%23new-answer', 'question_page');
                    }
                    );

                    Post as a guest















                    Required, but never shown





















































                    Required, but never shown














                    Required, but never shown












                    Required, but never shown







                    Required, but never shown

































                    Required, but never shown














                    Required, but never shown












                    Required, but never shown







                    Required, but never shown







                    Popular posts from this blog

                    Wiesbaden

                    To store a contact into the json file from server.js file using a class in NodeJS

                    Marschland